- Wed Oct 18, 2017 1:23 pm
#40669
Complete Question Explanation
(The complete setup for this game can be found here: lsat/viewtopic.php?t=15480)
The correct answer choice is (A)
This question is relatively easy if you identified that K can only be shown third or fourth. Because J must be shown sixth or seventh, J and K can never be consecutive, and answer choice (A) cannot be true and is therefore correct.
Note that answer choice (B) is proven possible by the hypothetical produced in question #1.
(The complete setup for this game can be found here: lsat/viewtopic.php?t=15480)
The correct answer choice is (A)
This question is relatively easy if you identified that K can only be shown third or fourth. Because J must be shown sixth or seventh, J and K can never be consecutive, and answer choice (A) cannot be true and is therefore correct.
Note that answer choice (B) is proven possible by the hypothetical produced in question #1.